Emergency and Acute cases - Clinic exit
8. Case 8 A 55-year-old man presents to the emergency department (ED) complaining of abdominal pain. The patient relates that he has been having intermittent pain throughout the abdomen for the past 12 hours, and since the onset of pain, he has vomited twice. His past medical history is significant for hypertension and colon cancer, for which he underwent laparoscopic right colectomy 8 months ago. The patient indicates that he has not had any recent abdominal complaints. His last bowel movement was 1 day ago, and he denies any weight loss and hematochezia. What physical exam do you want to do? What other diagnostic studies?
1. Abdominal exam - auscultate, percussion, palpation, Murphy's sign, McBurney's point, CVA tenderness, rebound tenderness 2. Cardiopulmonary exam 3. CBC, CMP (LFTs, amylase, lipase), 4. UA, FOT/FIT
Case 12 A 24-year-old woman presents to the emergency department (ED) with complaints of flank pain and fever for the last 1-2 days. She describes feeling pain with urination over the previous week. She is currently feeling febrile and nauseated but has not vomited. The pain in her right flank is a dull, constant, nonradiating ache that she rates as 5/10 for pain. She took 600 mg of ibuprofen last night to help her sleep, but this morning the pain persisted, so she came into the ED for evaluation. She reports that she is sexually active and that her last menstrual period was 1 week ago. She denies any vaginal discharge or abdominal pain. Her vital signs include a temperature of 38.3°C (101°F), heart rate of 112 beats per minute, respiratory rate of 15 breaths per minute, and blood pressure of 119/68 mm Hg. What physical examination do you want to perform? What are your next diagnostic steps?
1. Abdominal exam - auscultation, palpation, rebound tenderness, CVA tenderness 2. UA, CBC, CMP (LFTs, amylase, lipase, alk phos)
Case 19 A 26-year-old woman presents to the emergency department (ED) with a 6-hour history of worsening abdominal pain. She states the pain initially was a dull pain near her umbilicus but has since moved to her lower right side. She rates the pain as 8 on a scale of 10 and crampy in nature. The patient states that she noted some vaginal spotting this morning but denies any passage of clots or tissue. The patient ate breakfast that morning but states she has not eaten since because she feels nauseous. She denies any fever or chills or any change in her bowel habits. Upon further questioning, the patient states her last menstrual period was 2 months ago, but her periods are irregular. She also states that she was told that she had a vaginal infection a year ago but does not recall having been treated for the illness. On physical examination, her blood pressure is 120/76 mm Hg, heart rate is 105 beats per minute, and she is afebrile. What PE would you perform?
1. Abdominal exam - including Rovsing's sign, rebound tenderness, iliopsoas and obturator sign, McBurney's sign, Murphy's sign 2. Pelvic exam - cervical motion tenderness, adnexal tenderness, speculum exam - vaginal discharge 3. Cardiopulmonary exam 4. Labs - CBC, CMP, UA, pregnancy test (beta - HCG), referral for abdominal and transvaginal U/S
Case 16 A 19-year-old man is brought to the emergency department (ED) with an altered level of consciousness. His family reports that he has had diffuse abdominal pain and vomiting. The patient's symptoms began several days ago when he complained of "the flu." His symptoms at onset included profound fatigue, nausea, mild abdominal discomfort, and some urinary frequency. Today he was found in bed moaning, but otherwise he is unresponsive. His past medical history is unremarkable, and he is currently taking no medications. On examination, the patient appears pale and ill. His temperature is 36 °C (96.8 °F), pulse rate is 140 beats per minute, blood pressure (BP) is 82/40 mm Hg, and respiratory rate is 32 breaths per minute. What PE would you like to perform? What other diagnostic tests should be run?
1. Abdominal exam, cardiac exam, lung exam, HEENT (check mucous membranes, eyes for visual disturbances), Pysch (orientation, MME), skin exam (check for turgor) and neurologic exam (cranial nerve dysfunction and change in LOC). 2. Labs - CBC, CMP, blood glucose. UA and EKG
10. Case 18 An 18-year-old adolescent woman presents to the emergency department (ED) complaining of a 1-week history of abdominal pain. She tells you that she and her friends recently returned from spring break vacation in Mexico, and she has noticed a constant ache that is worse on her right side. What questions do you have for her?
1. Ask about water intake (bottled, tap, unknown), drinks, food? 2. Does the pain radiate anywhere? What is the character of the pain (superficial, deep, sharp, dull, crampy)? 3. Fever, Nausea, vomiting, diarrhea (bloody, mucus, tarry stools)? 4. Is there any correlation between food and the abdominal pain? 5. Muscle aches, pains, joint pain? 6. SOB? Cough, headache, fatigue? Change in mental status like confusion, loss of concentration? 7. Was she sexually active? What was date of last menses? (Remember the 5 P's - Partners, Prevention of pregnancy, protection from STDs, Practices, past history of STDs) 8. Does she have any vaginal discharge, unusual odor, abnormal bleeding?
Case 13 A 27-year-old woman reports that her mouth has been drooping in the right corner since yesterday. As a result, it is difficult for her to drink water without drooling. She cannot close her right eye completely, and her right eye is red and irritated. She denies having headaches, visual disturbances, nausea, or vomiting. She does not have any history of trauma. Her past medical history is unremarkable. She is not taking any medications. On physical examination, the right corner of her mouth droops, and the right nasolabial fold is absent. The right lower eyelid is sagging, and the patient cannot completely close her right eye. On attempts to close the right eye, the eye rolls upward. The patient also cannot wrinkle her forehead. The other cranial nerves seem to be normal, and the neurologic examination reveals no deficits other than as stated. What is on your ddx? What is your working diagnosis?
1. Bells palsy, Lyme disease, stroke, tumors of the temporal bone, Ramsey Hunt syndrome, acoustic neuromas, and malignant otitis externa, MS, Guillain-Barre disease, polio, sarcoid, and HIV 2. Working dx - Bell palsy 3. Treatment plan - Corticosteroids and follow up 4 weeks or sooner if symptoms worsen
Case 12 A 24-year-old woman presents to the emergency department (ED) with complaints of flank pain and fever for the last 1-2 days. She describes feeling pain with urination over the previous week. What other questions do you have for her?
1. Can you describe the pain? (Dull, sharp, cramping, deep or superficial), Does the pain radiate anywhere? 2. Severity of pain? 3. Alleviating factors? Aggravating factors? 4. Has she felt nausea or vomited? Any abdominal pain? 5. Has she ever had anything like this happen before? 6. Vaginal discharge, increased urgency, frequency, cloudy urine? Bloody urine? 7. Headaches, lightheadedness, dizziness 8. In a sexual relationship, sexually active, protection used? Menses? 9. PMhx: medications, supplements, history of UTI
Case 9 A 19-year-old woman is brought into the emergency department (ED) complaining of abdominal pain and diarrhea of 3-day duration. She has also been nauseous and has not been able to drink much liquid. Five days ago she returned from a camping trip in New Mexico but did not drink from natural streams. She denies fever but states that she has had some chills. Her stools have been watery, brown, and profuse. The patient denies health problems. What PE would you like to perform? Labs?
1. Cardiac exam (JVD)and pulmonary exam 2. HEENT - check conjunctiva, mucosa for pallor 3. Skin - check for capillary refill (cyanosis), lesions, tenting 4. Abdominal pain - auscultation, percussion and palpation, rebound tenderness 5. Rectal exam 6. Labs - CMP, CBC, Fecal occult blood test, UA, stool culture, pregnancy test 7.
Case 20 A 34-year-old man presents to the emergency department (ED) complaining of shortness of breath and right-sided chest pain that increases with deep breathing. He states that it started suddenly when he woke up and worsened with activity. He denies fever, chills, nausea, vomiting, or cough. He has a recent history of multiple gunshot wounds, resulting in ongoing pain in his upper back and T10 paraplegia. He quit smoking tobacco when he was hospitalized and denies any alcohol or illicit drug use. One week ago, he was discharged from the hospital to a rehabilitation facility. He is currently taking acetaminophen/hydrocodone and ibuprofen for his pain, which has increased with physical therapy and occupational therapy. He is also taking hydrochlorothiazide and lisinopril for hypertension and fluoxetine for depression. What PE would you like to perform? What diagnostic test will you do? What will you send out for referral?
1. Cardiac exam - JVD, peripheral pulses, edema, 2. Respiratory exam - auscultation, percussion, chest expansion 3. MSK exam - extremities swelling, redness, edema 4. Labs: D-dimer, CBC, CMP, 5. EKG, send for chest x-ray, CT scan
Case 4 A 25-year-old man presents to the emergency department (ED) with palpitations and lightheadedness. These symptoms started acutely about 1 hour prior to arrival while he was watching television. The patient does not have any chest pain or shortness of breath. He also denies any recent fever, upper respiratory symptoms, and hemoptysis. He does not have any significant past medical history or family history. He is not taking any medications, does not smoke, and has never used any illicit drugs. On examination, his temperature is 98.2°F, blood pressure is 88/46 mm Hg, heart rate is 186 beats per minute, respiratory rate is 22 breaths per minute, and oxygen saturation is 97% on room air. He is mildly anxious but otherwise in no acute distress. What physical exam will you do? What are your next diagnostic steps?
1. Cardiac exam, pulmonary exam, skin exam checking for edema and cyanosis, check peripheral pulses - brachial, radial, popliteal and pedal pulses 2. EKG, echo, chest x-ray 3. CBC, CMP, TSH, BNP, d-dimer 4. The physical examination will initially focus on assessing the patient's stability and adequacy of the airway, breathing, and circulation (ABCs). Any evidence of hypotension, pulmonary edema, acutely altered mental status, or ischemic chest pain indicates that the patient is unstable and that treatment must be initiated immediately (see treatment section later in chapter). Once the patient is stabilized, a complete head-to-toe examination can be performed. Special consideration should be given to the cardiovascular and pulmonary components of the examination: auscultating heart sounds for gallops, murmurs, and rubs, palpating for the point of maximal impulse and any heaves, inspecting for jugular venous distention, listening for any rales or other findings of volume overload, assessing the quality of peripheral pulses. The examination may also reveal clues regarding underlying causes of tachycardia (eg, pale mucous membranes with anemia, thyromegaly or goiter with thyrotoxicosis, barrel chest or nail clubbing with chronic lung disease).
Case 5 A 63-year-old woman arrives in the emergency department (ED) in respiratory distress. The paramedics who transported her were not able to obtain any information about her past medical history but did bring her bag of medications, which included furosemide. On examination, her temperature is 37.5°C (99.5°F), blood pressure is 220/112 mm Hg, heart rate is 130 beats per minute, respiratory rate is 36 breaths per minute, and oxygen saturation is 93% on a non-rebreather mask. What physical exam would you perform? What are your next diagnostic steps?
1. Cardio - JVD/JVP, auscultation, palpation apical impulses, percussion, peripheral pulses 2. Pulmonary - auscultation, 3. Abdominal exam - auscultation (bruits), palpation (hepatosplenomegaly), ascites (special maneuver) 4. Skin/MSK - edema, cool lower extremities, Beau's lines (nail ridging or depression), check for clammy skin, diaphoresis, delayed capillary refill and thready pulses 5. EKG, Chest x-ray, transthoracic echocardiography, Labs (BNP, troponin I, CKMB, CMP(BUN/creatinine), CBC, TSH, Iron panel)
Case 7 You are working in the emergency department (ED) of a 15-bed rural hospital without CT scan capabilities, and a 25-year-old, previously healthy woman presents for evaluation of abdominal pain. The patient describes her pain as having been present for the past 3 days. The pain is described as constant, exacerbated by movements, and associated with subjective fevers and chills. She denies any recent changes in bowel habits, urinary symptoms, or menses. Her last menstrual period was 6 days ago. The physical examination reveals temperature of 38.4°C (101.1°F), pulse rate of 110 beats per minute, blood pressure of 112/70 mm Hg, and respiratory rate of 18 breaths per minute. What physical exam do you want to perform? What diagnostic tests?
1. Cardiopulmonary exam 2. Abdominal exam - auscultation, palpation, Murphy's sign, McBurney's point, rebound tenderness (appendicitis or cholecystitis), Rosving (appendicitis) 3. Pelvic exam - check for discharge, cervical motion tenderness, adnexal tenderness, cervical friability, pelvic mass 4. UA, pregnancy test, labs (CBC, CMP, ESR, CRP), serum amylase/lipase, LFTs, 5. Abdominal and/or pelvic ultrasound
Case 18 An 18-year-old adolescent woman presents to the emergency department (ED) complaining of a 1-week history of abdominal pain. She tells you that she and her friends recently returned from spring break vacation in Mexico, and she has noticed a constant ache that is worse on her right side. The patient's mother is worried because her daughter has been unable to eat or drink anything for 2 days. After asking the mother to step out of the room while you examine the patient, she tells you that she has had five sexual partners, occasionally uses condom for birth control, and has never been pregnant. Her last menstrual period was 2 weeks ago and was heavier than normal. On physical examination, her blood pressure was 100/70 mm Hg, pulse 110 beats per minute, respirations 22 breaths per minute, and temperature 38.9°C (102.1°F). What PE would you like to perform? What diagnostic test?
1. Cardiopulmonary exam - 2. Abdominal exam - auscultation, percussion, palpation, Murphy's test, rebound tenderness, Rovsing's sign, obturator and iliopsoas muscle test 3. Pelvic exam - checking for vaginal discharge, adnexal and cervical motion tenderness, collect vaginal sample, bimanual exam, 4. Labs: CBC, CMP, UA, pregnancy test, wet mount, PCR tests for chlamydia, gonorrhea, HIV, referral for transvaginal ultrasound
Case 12 A 24-year-old woman presents to the emergency department (ED) with complaints of flank pain and fever for the last 1-2 days. She describes feeling pain with urination over the previous week. She is currently feeling febrile and nauseated but has not vomited. The pain in her right flank is a dull, constant, nonradiating ache that she rates as 5/10 for pain. She took 600 mg of ibuprofen last night to help her sleep, but this morning the pain persisted, so she came into the ED for evaluation. She reports that she is sexually active and that her last menstrual period was 1 week ago. She denies any vaginal discharge or abdominal pain. Her vital signs include a temperature of 38.3°C (101°F), heart rate of 112 beats per minute, respiratory rate of 15 breaths per minute, and blood pressure of 119/68 mm Hg. Her examination is significant for tenderness to palpation on her right costovertebral angle (CVA). What's your ddx? What's your working dx?
1. Cystitis, pyelonephritis, urethritis, vaginitis, ectopic pregnancy, cholecystitis, cholangitis, pancreatitis, appendicitis, colitis, and pneumonia. 2. Pyelonephritis 3. Summary: This otherwise healthy young woman presents with dysuria, flank pain, fever, and nausea. She is febrile, tachycardic, and has CVA tenderness. 4. Most likely diagnosis: Urinary tract infection (UTI) complicated by pyelonephritis. This should be confirmed with a urinalysis. 5. Treatment: Antibiotics, hydration, analgesia, antipyretics, and exclusion of other pathology. ND plan - antibiotics, probiotics to replenish gut microbiome after abx, cranberry juice or D-mannose to prevent adhesion of bacteria, increase water intake 2 L to flush out bacteria.
Case 18 An 18-year-old adolescent woman presents to the emergency department (ED) complaining of a 1-week history of abdominal pain. She tells you that she and her friends recently returned from spring break vacation in Mexico, and she has noticed a constant ache that is worse on her right side. The patient's mother is worried because her daughter has been unable to eat or drink anything for 2 days. After asking the mother to step out of the room while you examine the patient, she tells you that she has had five sexual partners, occasionally uses condom for birth control, and has never been pregnant. Her last menstrual period was 2 weeks ago and was heavier than normal. On physical examination, her blood pressure was 100/70 mm Hg, pulse 110 beats per minute, respirations 22 breaths per minute, and temperature 38.9°C (102.1°F). Her heart has a regular rate and rhythm without murmurs. Lungs are clear to auscultation bilaterally. The abdominal examination reveals a diffusely tender lower abdomen, greater on the right than left, and the patient exhibits voluntary guarding. Examination of the pelvis reveals a greenish, foul-smelling discharge with a red, friable-appearing cervix. Bimanual examination reveals an exquisitely tender cervix with fullness and pain in the right adnexal area. The wet prep of the discharge shows many white blood cells (WBCs), no clue cells, no trichomonas, and no Candida. A urine pregnancy test is negative. What is on your ddx and Wdx?
1. Ddx: Appendicitis, pancreatitis, gastritis (Giardia, Campylobacter, E. coli, Salmonella, etc), pregnancy, cholecystitis, ectopic pregnancy, STI, ovarian cysts (ruptured) a. Dynamed ddx: Ovarian cyst or torsion, endometriosis, fibroids, BV, cervicitis, nephrolithiasis, renal colic, UTI, diverticular abscess, IBS, gastroenteritis b. The differential diagnosis of acute PID includes appendicitis, ectopic pregnancy, endometriosis, ovarian torsion, hemorrhagic corpus luteum cyst, benign ovarian tumor, and inflammatory bowel disease. 2. Summary: An 18-year-old nulliparous adolescent woman complains of severe abdominal pain, vaginal discharge, fever, nausea, and vomiting. She displays cervical motion tenderness and her right adnexa appear to have some fullness and tenderness on examination. 3. Most likely diagnosis: Pelvic inflammatory disease (PID). 4. Next step: Transvaginal ultrasound to rule out tubo-ovarian abscess, assays for gonorrhea (GC) and Chlamydia (NAAT), complete blood count (CBC), and screen for sexually transmitted infections (STIs). 5. Next treatment step: Admit the patient and start IV antibiotic therapy. Treatment is usually parenteral. Outpatient management includes intramuscular ceftriaxone 250 mg once and oral doxycycline 100 mg twice daily for 14 days with or without metronidazole 500 mg twice daily for 14 days (Table 35-2). Patients should ideally be seen back in 48 hours to assess for improvement. Send to hospital if not better in 72 hours ND plan - F/U in 48-72 hours to assess for improvement, eat vegetable broth, lightly sauteed vegetables to maintain nutrients and help support immune system to clear infection, water w/electrolytes to rehydrate and replenish electrolytes, antioxidants (vitamin C 1 g/day), probiotics to be used after completing the round of abx given at the hospital to support gut microbiome or replenish.
Case 24 A 62-year-old man presents to your office for an acute visit because of coughing and shortness of breath. He is well known to you because of multiple office visits in the past few years for similar reasons. He has a chronic "smoker's cough" but reports that in the past 2 days his cough has increased, his sputum has changed from white to green in color, and he has had to increase the frequency with which he uses his albuterol inhaler. He denies having a fever, chest pain, peripheral edema, or other symptoms. His medical history is significant for hypertension, peripheral vascular disease, and two hospitalizations for pneumonia in the past 5 years. He has a 60 pack-year history of smoking and continues to smoke two packs of cigarettes a day. On examination, he is in moderate respiratory distress. His temperature is 98.4 °F, his blood pressure is 152/95 mm Hg, his pulse is 98 beats/min, his respiratory rate is 24 breaths/min, and he has an oxygen saturation of 94% on room air. His lung examination is significant for diffuse expiratory wheezing and a prolonged expiratory phase of respiration. There are no signs of cyanosis. The remainder of his examination is normal. A chest x-ray done in your office shows an increased anteroposterior (AP) diameter and flattened diaphragm, but otherwise he has clear lung fields. What is on your ddx? What is your wdx? What is your plan?
1. Ddx: Asthma, pneumonia, COPD (bronchitis, emphysema), allergic rhinitis, URI, heart failure 2. Wdx: Emphysema 3. Plan: 4. Summary: A 62-year-old man presents with, A long history of smoking, Dyspnea, coughing, and wheezing, Increased sputum production and a change in sputum character, 5. Most likely cause of dyspnea: Acute exacerbation of chronic obstructive pulmonary disease (COPD)., 6. Appropriate acute treatments: Antibiotic, bronchodilators, systemic corticosteroids. 7. Interventions to reduce future exacerbations: Smoking cessation, long-acting bronchodilator, inhaled corticosteroid, influenza, and pneumococcal polysaccharide vaccination. 8. The primary diagnostic test of lung function is spirometry. In normal aging, both the FVC and FEV1 reduce gradually and proportionally over time. In normal-functioning lungs, the ratio of the FEV1 to FVC is greater than 0.7. In COPD, both the FVC and FEV1 are reduced, and the ratio of FEV1 to FVC is less than 0.7, indicating an airway obstruction.
Case 14 A 50-year-old woman presents with a severe headache of abrupt onset of 10 hours' duration. The pain is diffuse, throbbing, and worsened when she went outside into the sunlight. She denies any recent fever, neck pain, numbness, weakness, vomiting, and any change in vision. She was concerned because she has never had a headache as strong as this before. Her past medical and family histories are unremarkable. She does not take any medications, does not smoke, and only drinks alcohol socially. On examination, her temperature is 36.9°C (98.4°F), blood pressure is 136/72 mm Hg, heart rate is 88 beats per minute, and respiratory rate is 16 breaths per minute. She is not in any acute distress but appears to be mildly uncomfortable. Her pupils are equal and reactive bilaterally. There is no evidence of papilledema on fundoscopic examination. Movement of her neck causes her some increased discomfort. Her neurological examination is normal, including cranial nerves, strength, light touch sensation, deep-tendon reflexes, and finger-to-nose. The computed tomography (CT) scan of her head is normal. What is on your ddx? What is your working dx? Plan?
1. Ddx: Giant cell arteritis (temporal arteritis), migraine, cluster headache, tension/stress headache, meningitis, stroke, dehydration, subarachnoid hemorrhage. Hypertensive encephalopathy, acute angle closure glaucoma 2. Subarachnoid hemorrhage 3. Summary: A 50-year-old woman presents with, A 10-hour history of the worst headache of her life, Photophobia and a normal neurologic exam, Normal findings on CT imaging of the head, 4. Most likely diagnosis: Subarachnoid hemorrhage (SAH). 5. Next diagnostic step: Lumbar puncture or CT angiogram (CTA). 6. the clinician should assess for neurologic function, mental status, and signs of central nervous system infection. It is critical that any "red flag symptoms" such as neurologic deficits, fever, immunocompromised state, or history of a clotting disorder be identified, any of these findings could point to a more serious condition. This 50-year-old woman has an acute onset of severe headache, described as the "worst headache of her life." The acute onset and severity of her symptoms are concerning for SAH. CT can miss SAH, and LP should be performed to evaluate for bloody or xanthochromic CSF and to assess for meningitis. That said, the clinician should not solely rely on imaging and lab studies. Clinical evaluation is most important.
Case 27 A 26-year-old woman presents to the emergency department complaining of bleeding from her nose and mouth that started last night. She also noticed small, reddish spots on her lower extremities when she got out of the bed this morning. She denies fever, chills, nausea, vomiting, abdominal pain, or joint pain. The patient reports she had developed an upper respiratory infection 2 weeks prior to the emergency department visit, but the infection resolved. She has no current medical problems. Her menses have been normal, and her last menstrual period was approximately 2 weeks ago. She denies excessive bleeding in the past. Prior to this episode, she never had epistaxis, easy bruising, or bleeding into her joints. There is no family history of abnormal bleeding. The patient does not take any medications. On examination, she is alert, oriented, and somewhat anxious. Her blood pressure (BP) is 110/70 mm Hg, her heart rate (HR) is 90 beats per minute (bpm), and she is afebrile. No pallor or jaundice is noted. There is bright red blood oozing from the nose and the gingiva. Skin examination reveals multiple 1-mm flat reddish spots on her lower extremities. The rest of the examination is normal. No lymphadenopathy or hepatosplenomegaly is noted. Her complete blood count (CBC) is normal, with the exception of a platelet count of 18,000/mm3. Prothrombin time (PT) and partial thromboplastin time (PTT) are normal. What is on your ddx? What is your wdx? Plan?
1. Ddx: Liver dz (including cirrhosis or portal htn) or renal dz, ethanol abuse, DIC, TTP, HUS, ITP, drug-induced thrombocytopenia, familial thrombocytopenia a. Differentiate causes of thrombocytopenia, specifically thrombocytopenic purpura versus other platelet disorders, such as thrombotic thrombocytopenic purpura (TTP), hemolytic uremic syndrome (HUS), and disseminated intravascular coagulation (DIC) 2. Wdx: 3. Plan 4. Summary: A 26-year-old woman presents with, Persistent epistaxis, Petechiae on legs, No past medical history and no personal or family history of excessive bleeding or easy bruising, No lymphadenopathy or hepatosplenomegaly, Other than thrombocytopenia (18,000/µL), CBC normal, Prothrombin time and PTT normal 5. Most likely diagnosis: Immune thrombocytopenic purpura (ITP). 6. Next diagnostic step: Peripheral smear to rule out other causes of thrombocytopenia. 7. Best initial treatment: Oral corticosteroids.
Case 22 A 3-year-old child presents with the complaint of right ear pain for 1 day. The patient's mother says that 3 days prior he developed a cough and yellow nasal discharge. One day prior, he began complaining of right ear pain and had a temperature of 101.5 °F (38.6 °C) at that time. He has had no nausea, vomiting, diarrhea, or rash. His urine output has been good, his solid intake is slightly decreased, but he is drinking well. She reports that he is fussy at night but is acting normal throughout the day. His mother notes that his pain is minimally improved with acetaminophen and ibuprofen. What is on your ddx? What is your wdx? What is your plan?
1. Ddx: Otitis media, acute otitis externa, mastoiditis, URI 2. Wdx: Acute otitis media 3. Plan: No medications at this time, the majority of cases self-resolve, enter a prescription for antibiotics in the event that he gets worse and if symptoms do not improve in 48 - 72 hours. Have parents follow up in 3-5 days, within a week to confirm that infection is clearing. Meantime take children Tylenol for the pain and onion ear muffs, warm compress on the ear are other options. 4. Summary: A 3-year-old boy presents with, Ear pain of 1 day's duration, Cough and yellow nasal discharge that began 3 days prior, Fever, Fussiness at night, No nausea, vomiting, diarrhea, or rash, Good urine output and liquid intake, slightly decreased solid intake 5. Most likely diagnosis: Acute otitis media (AOM). 6. Best therapy: Oral antibiotics. 7. In some situations, especially in a child older than 6 months with mild symptoms (ie, mild otalgia for less than 48 hours, temperature below 39 °C), a "watchful waiting" period of a few days may be indicated because many AOM cases self-resolve. Numerous studies have shown that only about one-third of children with evidence of AOM initially observed for a period had persistent or worsening symptoms that required rescue antibiotics. Alternatively, an effective approach is providing a "safety net prescription" to be filled if symptoms do not improve in 48 hours. This has also led to reduced antibiotic use and has been accepted by caregivers. Ensuring close medical follow-up is paramount if the choice is made to withhold antibiotics and to observe children with mild AOM.
Case 19 A 26-year-old woman presents to the emergency department (ED) with a 6-hour history of worsening abdominal pain. She states the pain initially was a dull pain near her umbilicus but has since moved to her lower right side. She rates the pain as 8 on a scale of 10 and crampy in nature. The patient states that she noted some vaginal spotting this morning but denies any passage of clots or tissue. The patient ate breakfast that morning but states she has not eaten since because she feels nauseous. She denies any fever or chills or any change in her bowel habits. Upon further questioning, the patient states her last menstrual period was 2 months ago, but her periods are irregular. She also states that she was told that she had a vaginal infection a year ago but does not recall having been treated for the illness. On physical examination, her blood pressure is 120/76 mm Hg, heart rate is 105 beats per minute, and she is afebrile. In general, she is in mild distress. The abdomen reveals tenderness to palpation in her right lower quadrant that is greater than that in the left lower quadrant. The examination reveals some minimal voluntary guarding, but no rebound tenderness is appreciated. On pelvic examination, the uterus appears mildly enlarged without cervical motion tenderness. There are no masses or tenderness in the adnexal region. Her complete blood count (CBC) reveals a mildly elevated white blood cell count with a left shift. A beta-human chorionic gonadotropin (β-hCG) was 4700 mIU/mL. A transvaginal sonogram reveals an empty uterus, and no adnexal masses or free fluid is noted. What is your ddx? What's your working diagnosis?
1. Ddx: PID, ovarian torsion, pancreatitis, appendicitis, ectopic pregnancy, UTI, adnexal mass, nephrolithiasis, cholecystitis, IBD, PUD, diverticulitis, colitis, a. The differential diagnosis of ectopic pregnancy includes many other gynecologic and surgical illnesses. Most common are salpingitis, threatened or incomplete abortion, ruptured corpus luteum, adnexal torsion, and appendicitis. The diagnosis of ectopic pregnancy must be considered in any woman of reproductive age with abnormal vaginal bleeding and abdominal pain. 2. Summary: A 26-year-old woman complains of severe abdominal pain, nausea, and vaginal spotting. She has a positive pregnancy test, a quantitative β-hCG level of 4700 mIU/mL, and a transvaginal sonogram showing no intrauterine pregnancy. 3. Most likely diagnosis: Ectopic pregnancy. 4. Next step: Diagnostic versus operative laparoscopy. 5. Initial treatment: Establish an IV line and stabilize the patient in preparation for surgery. ND plan: Follow up in 2 -4 weeks, discuss contraceptives, healthy sex practices, counseling for possible depression or anxiety regarding, referral to pelvic floor therapy to prevent adhesions
Case 16 A 19-year-old man is brought to the emergency department (ED) with an altered level of consciousness. His family reports that he has had diffuse abdominal pain and vomiting. The patient's symptoms began several days ago when he complained of "the flu." His symptoms at onset included profound fatigue, nausea, mild abdominal discomfort, and some urinary frequency. Today he was found in bed moaning, but otherwise he is unresponsive. His past medical history is unremarkable, and he is currently taking no medications. On examination, the patient appears pale and ill. His temperature is 36 °C (96.8 °F), pulse rate is 140 beats per minute, blood pressure (BP) is 82/40 mm Hg, and respiratory rate is 32 breaths per minute. He has dry mucous membranes and sunken eyes, there is an unusual odor to his breath. Lungs are clear bilaterally with regular, deep respirations. Cardiac examination reveals tachycardia, no murmurs, rubs, or gallops. The abdomen is diffusely tender to palpation, with hypoactive bowel sounds and involuntary guarding. Rectal examination is normal. Skin is cool and dry with decreased turgor. On neurologic examination, the patient moans and localizes pain but does not speak coherently. Laboratory studies show a leukocyte count of 16,000 cells/mcL and normal hemoglobin and hematocrit levels. Electrolyte levels include a sodium of 124 mEq/L, potassium of 3.4 mEq/L, chloride of 98 mEq/L, and bicarbonate of 6 mEq/L. Blood urea nitrogen (BUN) and creatinine are moderately elevated. Serum glucose is 740 mg/dL (41.1 mmol/L). The lipase, bilirubin, aspartate transaminase (AST), alanine transaminase (ALT), and alkaline phosphatase are within normal limits. A 12-lead electrocardiogram (ECG) shows only sinus tachycardia. His chest x-ray (CXR) is normal. What is on your ddx? What is your working dx? What is your plan?
1. Ddx: Pancreatitis, appendicitis, diverticulitis, DKA, HHS 2. Working dx: DKA 3. Send to ED - CMP, CBC, checking bicarbonate, anion gap, potassium, give chest x-ray. Give IV fluids 1st line treatment, replace potassium then give IV insulin. 4. Severe hyperglycemia and anion-gap acidosis, Altered level of consciousness, Reports by family of patient having diffuse abdominal pain and vomiting, Findings of dehydration, hyponatremia, and hypotension 5. Most likely diagnosis: Diabetic ketoacidosis (DKA). 6. Next steps: Management of the ABCs (airway, breathing, and circulation), including fluid resuscitation, the initiation of insulin therapy, and a careful search for any precipitating or concomitant illness
Case 7 You are working in the emergency department (ED) of a 15-bed rural hospital without CT scan capabilities, and a 25-year-old, previously healthy woman presents for evaluation of abdominal pain. The patient describes her pain as having been present for the past 3 days. The pain is described as constant, exacerbated by movements, and associated with subjective fevers and chills. She denies any recent changes in bowel habits, urinary symptoms, or menses. Her last menstrual period was 6 days ago. The physical examination reveals temperature of 38.4°C (101.1°F), pulse rate of 110 beats per minute, blood pressure of 112/70 mm Hg, and respiratory rate of 18 breaths per minute. Her skin is nonicteric. Cardiopulmonary examination is unremarkable. The abdomen is mildly distended and tender in both right and left lower quadrants. Involuntary guarding and localized rebound tenderness are noted in the right lower quadrant. The pelvic examination reveals no cervical discharge, cervical motion tenderness and right adnexal tenderness are present. The rectal examination reveals no masses or tenderness. Laboratory studies reveal white blood cell (WBC) count of 14,000 cells/mm3, a normal hemoglobin, and a normal hematocrit. The urinalysis reveals 3-5 WBC/high-power field (HPF), few bacteria, and trace ketones. What is on your ddx, Wdx and other work up?
1. Ddx: Pregnancy, ectopic pregnancy, PID, STI, appendicitis, pancreatitis, colitis, PUD, IBD, SIBO, diverticulitis Plan: Send to urgent care/ED for transvaginal U/S to r/o ovarian torsion/cyst, CT scan to r/o appendicitis ND: F/U in 72 hours a. Eat lightly, emphasizing vegetable broths, cooked vegetables, fruits, and culinary herbs, to add key immune supportive nutrients and antioxidants b. Vitamin C - additional immune and antioxidant support c. Acidophilus yogurt 4-8oz/day to prevent GI distress and vulvovaginal candida due to abx c. Discuss healthy sex practices, barrier methods 2. Summary: A 25-year-old, previously healthy woman presents with a 3-day history of lower abdominal pain and subjective fever. Her examination indicates the presence of fever and lower abdominal tenderness (right > left). The rectal examination is unremarkable. Her laboratory studies indicate leukocytosis. 3. Most likely diagnoses: Likely diagnoses include complicated acute appendicitis, pelvic inflammatory disease (PID), ovarian torsion, or other pelvic pathology. 4. Confirmatory studies: Begin with pregnancy test and pelvic ultrasonography to evaluate for possible ovarian and pelvic pathology. If these suggest pelvic source of pathology, then strong consideration should be given to perform exploratory laparoscopy or laparotomy.
Case 22 A 3-year-old child presents with the complaint of right ear pain for 1 day. The patient's mother says that 3 days prior he developed a cough and yellow nasal discharge. One day prior, he began complaining of right ear pain and had a temperature of 101.5 °F (38.6 °C) at that time. He has had no nausea, vomiting, diarrhea, or rash. His urine output has been good, his solid intake is slightly decreased, but he is drinking well. She reports that he is fussy at night but is acting normal throughout the day. His mother notes that his pain is minimally improved with acetaminophen and ibuprofen. His right ear is protruding with erythema and swelling, and postauricular tenderness. What is on your ddx? What is your wdx? What is your plan?
1. Ddx: acute otitis media, otitis media w/effusion, otitis externa, mastoiditis, postauricular cellulitis, erysipelas, insect bite 2. Wdx: Mastoiditis 3. Plan: send to the ED for CT scan
Case 25 A 45-year-old man presents to the clinic with a cough productive of purulent sputum of 3 weeks' duration. He says that he had just gotten over a cold a few weeks prior to this episode. He occasionally has fevers, and he coughs so much that he has chest pain. He reports having a mild sore throat and nasal congestion. He has no history of asthma or any chronic lung diseases. He denies nausea, vomiting, diarrhea, and any recent travel. He denies any smoking history. On examination, his temperature is 98.6 °F (37.0 °C), pulse is 96 beats/min, blood pressure is 124/82 mm Hg, respiratory rate is 18 breaths/min, and oxygen saturation is 99% on room air. Head, ears, eyes, nose, and throat (HEENT) examination reveals no erythema of the posterior oropharynx, tonsillar exudates, uvular deviations, or significant tonsillar swelling. Neck examination is negative. The chest examination yields occasional wheezes, but normal air movement is noted. What is on your ddx? What is your working dx? What is your plan?
1. Ddx: pneumonia, sinusitis, common cold, tuberculosis, pertussis, upper airway cough syndrome (post nasal drip), asthma, chronic bronchitis, bronchiectasis, heart failure, 2. Wdx: Acute bronchitis 3. Plan 4. Summary: A 45-year-old man presents with, No history of lung disease or smoking, 3 weeks of productive cough following an upper respiratory infection, History of occasional fever and coughing so much that he has chest pain, Normal HEENT examination, Occasional wheezes but normal air movement on chest examination 5. Most likely diagnosis: Acute bronchitis. 6. Next step: Bronchodilators, analgesics, and antitussives. Antibiotics have not been consistently shown to be beneficial. The illness is usually self-limited. 7. Common noninfectious causes of cough: Asthma, chronic obstructive pulmonary disease (COPD), malignancy, postnasal drip, gastroesophageal reflux disease (GERD), medication side effect (eg, angiotensin-converting enzyme inhibitors), congestive heart failure.
Case 11 A 54-year-old man is brought to the emergency department with complaints of generalized weakness, nausea, and nonspecific feelings of illness. The symptoms have progressed insidiously over 2-3 days. His past medical history is remarkable for longstanding diabetes and poorly controlled hypertension (HTN). He is currently taking many medications that include oral hypoglycemics, a diuretic, and an angiotensin-converting enzyme inhibitor (ACEI). On physical examination, the patient appears lethargic and ill. His temperature is 36.0°C (96.8°F), pulse rate is 70 beats per minute, blood pressure is 154/105 mm Hg, and respiratory rate is 22 breaths per minute. Head and neck examination shows normal conjunctiva and mucous membranes. There is moderate jugular venous distention, and the lungs have minor bibasilar rales. The cardiac examination reveals normal rate, no murmurs or rubs, and a positive S4. The abdomen is soft and nontender, with hypoactive bowel sounds and no organomegaly. Rectal examination is normal. Skin is cool and dry. Extremities demonstrate pitting edema to the knees bilaterally. On neurologic examination the patient moans and weakly localizes pain. He is oriented to person and place but cannot provide any further history. The initial rhythm strip is shown in Figure 23-1. What is your ddx? What's your working diagnosis?
1. Ddx: pseudohyperkalemia, hypoglycemia, stroke, assess for risk factors (CKD, acute renal faiure, ketoacidosis, meds that increase K+, obstructive uropathy) 2. Most likely diagnosis: Hyperkalemia - due to end-stage renal failure (aka chronic kidney disease) due to longstanding HTN and diabetes. 3. Next step: Management of the ABCs (airway, breathing, and circulation), including immediate vascular access and continuous cardiac monitoring, rapid stepwise administration of medication to reverse the effect of excess potassium (calcium), shift potassium into cells (insulin, sympathomimetics, and possibly sodium bicarbonate), and remove potassium from the body (sodium polystyrene sulfonate or diuretics). Arrange for emergency dialysis, and admit to the hospital. ND plan: Send to ED, F/U 48-72 hours, Low sodium diet, Diet (focus on plant-based proteins, low carb, lower iron), nephroprotective herbs, adaptogens for stress, exercise.
Case 17 A 46-year-old previously healthy woman presents with fever, nausea, vomiting, which began the last few days. What questions do you have for her?
1. Did anything happen prior to symptoms went out to eat different place? Travel, camping, around anyone else who is sick? 2. Muscle aches, abdominal pain/discomfort, diarrhea? 3. Back pain, flank pain, joint pain, headaches? 4. Change in urination increase urgency or frequency? 5. Does anything alleviate symptoms? Does anything make them worse? 6. SOB? Palpitations, chest pain, radiating pain? 7. Changes in mental status, confusion, decrease in concentration, loss of memory? 8. PMhx - medications, supplements, surgeries, 9. Fhx? Shx? Recreational drugs, alcohol use
Case 14 A 50-year-old woman presents with a severe headache of abrupt onset of 10 hours' duration. What are your questions for her?
1. Did anything happen prior to the HA 2. Could she describe the pain? Severity of the pain? 3. The location of the pain? Character of the pain? 4. Has anything like this occurred before? Is this HA different from other headaches? 5. Does she have any visual changes such as blurry vision, double vision? 6. Has she felt lightheaded/dizziness, syncope, weakness, numbness? 7. Does she have fever/chills, N/V, stiff neck? 8. Any confusion, altered mental status? 9. Anything make it better? NSAIDs, anything make it worse? 10. PMhx: medications, supplements 11. Shx: recreational drugs, alcohol use, tobacco 12. Fhx: CV, stroke,
Case 13 A 27-year-old woman reports that her mouth has been drooping in the right corner since yesterday. What questions do you want to ask her?
1. Did anything happen prior to this occurring? Did she have an accident or trauma? 2. Has she previously been ill? Has she had a stressful event? 3. Has she recently been camping in a wooded area? Traveling? 4. Did she feel any pain prior to this? Pain behind or in front of the ear? 5. Does she have difficulty eating, drinking, loss of sensation? 6. Does she have a fever/chills, headache, N/V? 7. Has she tried anything that has been helpful? 8. PMhx: medications, supplements, surgeries
Case 23 A 28-year-old man presents to your office complaining shortness of breath and wheezing for 6 hours. These symptoms have been progressively worsening. His past medical history is significant for seasonal allergies to pollen and ragweed. His family history is significant only for hypertension in both parents. His siblings and children are in good health without allergies or respiratory illness. He does not smoke or use illicit drugs and only drinks alcohol rarely. He has worked as an animal laboratory technician for the last 6 months. On questioning, his symptoms were initially more severe toward the end of the work week but are now continuous. He has been taking albuterol and ICS. He states that he is short of breath at rest. He is agitated and sitting with his hands on his knees. He's speaking in single words and there is accessory muscle use. On examination, he is afebrile, his body mass index is 23 kg/m2, blood pressure is 120/75 mm Hg, pulse is 125 beats/min, and respiratory rate is 33 breaths/min. SpO2 is 89% on room air. His conjunctivae are injected, there is mild clear ocular discharge, and his nasal turbinates are boggy without visible polyps. His lung examination reveals a prolonged expiratory-to-inspiratory ratio and end-expiratory wheezing at the bilateral bases. His heart examination is unremarkable, and there is no peripheral edema. What is your ddx? What is your wdx? What is your plan?
1. Exacerbation of asthma, bronchitis, COPD (emphysema, chronic bronchitis) 2. Wdx: exacerbation of asthma 3. Plan: Repeat SABA dosing, requires ER visit for nebulizer, supplemental O2, possible IV steroids
Case 5 A 63-year-old woman arrives in the emergency department (ED) in respiratory distress. The paramedics who transported her were not able to obtain any information about her past medical history but did bring her bag of medications, which included furosemide. On examination, her temperature is 37.5°C (99.5°F), blood pressure is 220/112 mm Hg, heart rate is 130 beats per minute, respiratory rate is 36 breaths per minute, and oxygen saturation is 93% on a non-rebreather mask. If she was not in such respiratory distress, what questions would you ask her?
1. Fatigue, fever, SOB, decreased exercise tolerance, orthopnea (SOB laying down), paroxysmal nocturnal dyspnea (PND) or shortness of breath? 2. SOB when bending forward 3. Chest pain or pressure? Palpitations? Lightheadedness? Syncope? 4. Abdominal discomfort, poor appetite, nausea and feeling bloated? 5. PMhx: medications, supplements, previous conditions MI, CAD, 6. Fhx: Sudden death, dilated cardiomyopathy, syncope, pacemaker or defibrillator 7. Shx: recreational drugs, alcohol use, smoking
7. Case 10 A 17-year-old adolescent male arrives at the emergency department (ED) after he developed an acute onset of severe right testicular pain about 4 hours ago while at soccer practice. The patient does not recall any recent trauma to the area and denies any fever, dysuria, or penile discharge. Although he has nausea, he does not have any abdominal pain or vomiting. On examination, his temperature is 99.5°F, blood pressure is 138/84 mm Hg, heart rate is 104 beats per minute, and respiratory rate is 22 breaths per minute. What physical exam do you want to do? What are your next diagnostic steps?
1. Genital exam - check for redness, swelling, tenderness, penile discharge or rash, cremasteric reflex, inguinal lymphadenopathy or hernia 2. Abdominal exam - auscultation, palpation 3. UA and doppler ultrasound
Case 25 A 45-year-old man presents to the clinic with a cough productive of purulent sputum of 3 weeks' duration. He says that he had just gotten over a cold a few weeks prior to this episode. He occasionally has fevers, and he coughs so much that he has chest pain. He reports having a mild sore throat and nasal congestion. He has no history of asthma or any chronic lung diseases. He denies nausea, vomiting, diarrhea, and any recent travel. He denies any smoking history. What PE do you want to perform? What other diagnostic steps do you want to take?
1. HEENT 2. Respiratory exam 3. Skin
Case 11 A 54-year-old man is brought to the emergency department with complaints of generalized weakness, nausea, and nonspecific feelings of illness. The symptoms have progressed insidiously over 2-3 days. His past medical history is remarkable for longstanding diabetes and poorly controlled hypertension (HTN). He is currently taking many medications that include oral hypoglycemics, a diuretic, and an angiotensin-converting enzyme inhibitor (ACEI). What PE do you want to perform? What are your next diagnostic steps?
1. HEENT - JVD, thyroid, check conjunctiva and mucous membranes (checking for dehydration) 2. Cardiovascular exam - auscultation, palpation, percussion, peripheral edema, peripheral pulses 3. Pulmonary exam - checking for wheezing, crackles, rales 4. Abdominal exam - auscultation (bruits), palpation (check for ascites, hepatosplenomegaly), check for decreased bowel sounds 5. Neuro exam - check reflexes and muscle strength 6. Psych - MME or MOCA, 7. EKG, chest x-ray 8. Labs (CBC, CMP, TSH, CRP, ESR), blood glucose, BNP, troponin I, (BUN, creatinine, serum lipase, bilirubin, AST, ALT and alkaline phosphatase)
Case 13 A 27-year-old woman reports that her mouth has been drooping in the right corner since yesterday. As a result, it is difficult for her to drink water without drooling. She cannot close her right eye completely, and her right eye is red and irritated. She denies having headaches, visual disturbances, nausea, or vomiting. She does not have any history of trauma. Her past medical history is unremarkable. She is not taking any medications. What physical examination should be done? Labs?
1. HEENT - check ear for vesicles due to Raymond-Hunt syndrome 2. Neuro exam - cranial nerve examination (close eyes, raise eyebrows, smile, puff out cheeks) 3. Skin exam - for rashes, bulls eye lesion (Lyme disease as a cause), skin lesions or blisters on the face
Case 17 A 46-year-old previously healthy woman presents to the emergency department (ED) via ambulance with altered mental status. According to her family, she has become increasingly confused over the last few days. Her symptoms started with fever, nausea, vomiting, and diarrhea and now include heart palpitations and confusion. She denies alcohol, tobacco, and illicit drug use. She also denies sick contacts and recent travel. On examination, her vital signs include a blood pressure of 160/85 mm Hg, heart rate of 140 beats per minute, respiratory rate of 28 breaths per minute, temperature of 40 °C (104 °F), and pulse oximetry of 96% on room air. What PE would you perform? What diagnostic tests should you do?
1. HEENT - check neck JVD and thyroid 2. Cardio exam - auscultation, peripheral pulses, check for edema 3. Respiratory exam 4. Neuro/Psych exam - MME, orientation 5. Labs - thyroid panel, CBC, CMP, `UA, EKG, pregnancy test
Case 21 A 63-year-old woman presents to the emergency department (ED) with severe left eye pain, redness, and blurred vision for 3 hours. Her right eye is asymptomatic. She denies preceding trauma, photophobia, ocular discharge, increased tearing, prior similar events, or past eye surgery. She is farsighted and sometimes wears nonprescription reading glasses. Other symptoms include seeing colored halos around the light fixtures in the ED, having a headache over her left brow, some nausea, and one episode of vomiting. She denies dizziness, weakness, imbalance, abdominal pain, and chest pain. On examination, her blood pressure is 138/80 mm Hg, and the other vital signs are normal. What PE do you want to perform? What other diagnostic tests doe you want to do?
1. HEENT - look at conjunctiva, fundoscopic exam, PERRLA, palpation
Case 22 A 3-year-old child presents with the complaint of right ear pain for 1 day. The patient's mother says that 3 days prior he developed a cough and yellow nasal discharge. One day prior, he began complaining of right ear pain and had a temperature of 101.5 °F (38.6 °C) at that time. He has had no nausea, vomiting, diarrhea, or rash. His urine output has been good, his solid intake is slightly decreased, but he is drinking well. She reports that he is fussy at night but is acting normal throughout the day. His mother notes that his pain is minimally improved with acetaminophen and ibuprofen. What PE would you perform? Any other diagnostic testing?
1. HEENT - otoscopic exam, palpation of ear and mastoid, check hearing by asking child questions, pneumatic otoscopy (check movement of tympanic membrane) 2. Check cervical lymph nodes
Case 27 A 26-year-old woman presents to the emergency department complaining of bleeding from her nose and mouth that started last night. She also noticed small, reddish spots on her lower extremities when she got out of the bed this morning. She denies fever, chills, nausea, vomiting, abdominal pain, or joint pain. The patient reports she had developed an upper respiratory infection 2 weeks prior to the emergency department visit, but the infection resolved. She has no current medical problems. Her menses have been normal, and her last menstrual period was approximately 2 weeks ago. She denies excessive bleeding in the past. Prior to this episode, she never had epistaxis, easy bruising, or bleeding into her joints. There is no family history of abnormal bleeding. The patient does not take any medications. On examination, she is alert, oriented, and somewhat anxious. What PE do you want to perform? What diagnostic tests should be done?
1. HEENT exam - exam nose, mouth for purpura in oral cavity 2. Respiratory exam 3. Skin exam: look for petechiae, purpura and ecchymoses 4. Lymph exam: examine for lymphadenopathy 5. Abdominal exam: checking for hepatosplenomegaly 6. Labs: CBC (anemia, infection as cause secondary ITP), peripheral blood smear (to check for destruction of platelets), HIV and Hepatitis C testing (secondary ITP)
Case 27 A 26-year-old woman presents to the emergency department complaining of bleeding from her nose and mouth that started last night. What questions do you want to ask her?
1. Has anything like this ever happened before? Has she had nose bleeds as a child? 2. Has she been traveling? What happened prior to symptoms, injury or trauma? 3. Does she have easy bruising, bleeding that won't stop? 4. Does she have bloody stool, change in BM? Change in urination, bloody urine? 5. Does she have any fever, nausea, vomiting, headache? 6. Does she have any SOB, cough, mucus? 7. Does she have any abdominal pain, back pain, joint pain? 8. When was her last menstrual period? Does she have heavy bleeding? How many tampons or pads does she use in a day? 9. PMhx: medications, supplements, surgeries, procedures, hx of autoimmune dysfunction 10. FHx: abnormal bleeding 11. Shx: recreational drugs, smoking, alcohol use, toxin exposure, needle stick
Case 20 A 34-year-old man presents to the emergency department (ED) complaining of shortness of breath and right-sided chest pain that increases with deep breathing. He states that it started suddenly when he woke up and worsened with activity. What questions do you have for him?
1. Has he been ill recently, recent travel? Recent trauma, injury? 2. Does he have SOB at rest? Unusual fatigue? 3. Does he have palpitations, chest pressure? Back pain? 4. Does he have a cough, mucus, sputum, blood? Has he noticed wheezing? 5. Fever, nausea, vomiting, headache, dizziness, lightheadedness, LOC? 6. Confusion, loss of concentration? 7. Back pain? Pain anywhere else - arm, leg pain? 8. Has he tried anything that makes the SOB better? Does anything make it worse? 9. PMhx: medications, supplements, history of respiratory or cardiac disease, surgery, procedures, hospital stays 10. SHx: recreational drugs, smoking, alcohol use 11. Fhx: significant history of cardiovascular dz, respiratory dz, cancer
Case 23 A 28-year-old man presents to your office complaining of a 3-month history of rhinorrhea, itchy eyes, and exertional cough and wheezing. These symptoms have been progressively worsening over the past few months. What questions do you have for him?
1. Has he ever had anything like this occur before? 2. Does he have allergies? What occurred 3 months ago? Was he ill? Was his family ill? 3. Does he notice any triggers (pollen, dust, mold, certain places)? Does anything help? 4. Is his cough dry or does he have mucus? 5. Does he have SOB at rest? Does he have SOB at night, requires several pillows? 6. Does he have any rashes, dry skin, hives etc? 7. Does he have chest pain, pressure, palpitations? 8. PMhx: medications, supplements, history of illnesses, 9. FHx: respiratory illness, autoimmune dz 10. Shx: home environment (basement apt, carpet, mold), work environment (toxins, dust, mold), smoking, recreational drugs, alcohol,
Case 15 A 57-year-old man presents to the emergency department (ED) with a 1-month history of worsening low back pain. What are your questions?
1. Has he injured his back? Trauma, MVA? What happened prior to the back pain? 2. Where is the pain located? 3. What is the quality of the pain? Sharp, pain, dull, localized, does it radiate? 4. Duration? Is it intermittent, constant, does it wake you up at night? 5. Fever, chills, weight loss, malaise? 6. Weakness in extremities, joint pain, urinary or fecal incontinence? 7. Alleviating factors? Aggravating factors? 8. PMhx: medications, supplements, surgeries, procedures 9. Fhx: Cancer, 10. Shx: Job,
Case 21 A 63-year-old woman presents to the emergency department (ED) with severe left eye pain, redness, and blurred vision for 3 hours. Her right eye is asymptomatic. What are your questions for her?
1. Has she had any trauma or injury to the eye? 2. Eye discharge, blurry/cloudy vision, double vision, increase in tears? Is she seeing colored halos around the lights? 3. What is the character of pain? Sharp, deep, dull? 4. How does she rate the severity of the pain? 0-no pain, 1-4 needs Advil or Tylenol, 5-7- need prescription medication, 8-10 needs to go to the ED 5. Has anything like this happened before? 6. Does she have any pain in her sinuses, nose pain? 7. Does she have any fever, nausea, vomiting, headache? Headache, frontal, temporal, occipital? 8. Has she tried anything that makes it better? Anything that makes it worse? 9. Has she had any lightheadedness, dizziness, syncope? 10. Has she had any confusion, loss of concentration? Mental changes? 11. Chest pain, palpitations, abdominal pain? 12. PMhx: medications, supplements, surgeries, procedures 13. SHx: recreational drugs, smoking, alcohol use 14. FHx: eye issues
Case 22 A 3-year-old child presents with the complaint of right ear pain for 1 day. What questions do you have for their parents?
1. Has the child injured themselves, fallen, MVA? 2. Has he recently been ill? Has anyone else in the home been ill or have similar symptoms? 3. Have they noticed any discharge from his ear? Has there been any hearing loss? Change in his hearing? 4. Have they been tugging, rubbing or holding their ear? 5. Are there any other painful areas? Head, eyes, discharge from eyes? 6. Fever, nausea, vomiting, lethargic, irritable? 7. Does he have nasal congestion, runny nose? Coughing? 8. Has he had any neck pain? 9. Does he have a rash or any other skin changes? 10. Have they tried anything that has been helpful? Does anything make it worse? 11. Has there been any changes in their eating or drinking habits? Do they have any abdominal pain? 12. Has he had difficulty sleeping, waking in the night with pain? 13. PMhx: medications, supplements, surgeries, procedures, immunizations 14. Fhx: history of otitis media 15. Shx: daycare, exposure to second hand smoke
Case 17 A 46-year-old previously healthy woman presents to the emergency department (ED) via ambulance with altered mental status. According to her family, she has become increasingly confused over the last few days. Her symptoms started with fever, nausea, vomiting, and diarrhea and now include heart palpitations and confusion. She denies alcohol, tobacco, and illicit drug use. She also denies sick contacts and recent travel. On examination, her vital signs include a blood pressure of 160/85 mm Hg, heart rate of 140 beats per minute, respiratory rate of 28 breaths per minute, temperature of 40 °C (104 °F), and pulse oximetry of 96% on room air. She is diaphoretic, agitated, and confused. Her neck exam reveals a prominent goiter. Her pulse is irregularly irregular. She has a fine tremor in her hands. What is on your ddx? What's the working diagnosis?
1. Heart failure, atrial fibrillation, acute liver failure, heatstroke, drug intoxication, hyperthyroidism, meningitis, pregnancy, pyelonephritis, infection 2. Thyroid storm (thyrotoxicosis) 3. An acutely altered mental status 4. A prominent goiter 5. Abnormal vital signs, including fever, tachycardia, tachypnea, and hypertension 6. No known medical conditions or drug and alcohol use 7. Most likely diagnosis: Thyroid storm. 8. Laboratory studies suggestive of the condition: Decreased serum thyroid-stimulating hormone (TSH) and elevated free T3 and T4 levels. 9. Best management: Beta-blockers (propranolol, esmolol), anti-thyroid medication (methimazole or propylthiouracil [PTU]), iodine, and glucocorticoids, given sequentially.
12. Case 3 A 73-year-old woman presents to the emergency department complaining of palpitations and new shortness of breath with minor exertion for the past 2 weeks. What are your questions? What physical exam will you do?
1. How frequently does she have palpitations? When does she notice the palpitations? 2. Can she describe palpitations? Severity? 3. Is she experiencing any chest pain or pressure? 4. Is she experiencing SOB at rest? Nighttime symptoms - sleeping w/additional pillows 5. Any lightheadedness/dizziness? 6. Is she experiencing an increase in Fatigue? 7. PMhx: DM, HTN, hyperlipidemia, sleep apnea, medications, supplements 8. Famhx: Cardiac disease 9. Shx: recreational drugs, alcohol use 10. PE: Cardiac exam - listen heart, check PMI(), check JVD, Pulmonary exam: listen to lungs, chest expansion, Check head and neck (JVP, elevated JVP w/HF), extremities for edema 11. Run EKG
Case 24 A 62-year-old man presents to your office for an acute visit because of coughing and shortness of breath. What questions do you have for him?
1. Is the cough productive? Mucus, dry, bloody? 2. Does he have wheezing or chest tightness? 3. What color is mucus? How much? Does have a smell? 4. Does he have fever, nausea or vomiting? 5. Does he have fatigue, weight loss, muscle loss? 6. When does he have SOB - exercise, pollen, dust? At rest, night, does he require several pillows? 7. Has he ever had anything like this occur before? 8. Does he have allergies? What occurred 3 months ago? Was he ill? Was his family ill? 9. Does he notice any triggers (pollen, dust, mold, certain places)? Does anything help? 10 duplicate - add another ? 11. duplicate 12. Does he have any rashes, dry skin, hives etc? 13. Does he have chest pain, pressure, palpitations? 14. PMhx: medications, supplements, past history allergies, asthma, respiratory conditions, cardiac HTN, hyperlipidemia 15. FHx: respiratory illness, autoimmune dz 16. Shx: home environment (basement apt, carpet, mold), work environment (toxins, dust, mold), smoking, recreational drugs, alcohol,
Case 15 A 57-year-old man presents to the emergency department (ED) with a 1-month history of worsening low back pain. The pain radiates down the back of both legs and suddenly increased yesterday. For the past 2 days, the patient has been having difficulty voiding and has had "to force the urine out." He has also noticed that the skin around his anus feels numb when he wipes with toilet tissue. He works in a warehouse but has been on light duty for the past month due to his back pain. He denies prior trauma to or surgery on his back. What PE would you want to do?
1. MSK exam, inspect, palpate, reflexes, muscle strength test 2. Neuro exam - check weakness, sensory loss
Case 4 A 25-year-old man presents to the emergency department (ED) with palpitations and lightheadedness. What are your questions?
1. Onset? What was he doing prior? Duration - constant, intermittent? 2. Has he felt faint or has he fainted? 3. Shortness of breath? Fever? Cough? 4. Extreme sports? Dehydration? 5. Has he been ill? Has anyone around him been ill? 6. Any other sx? Nausea or vomiting? 7. Has he been out of the country? 8. PMhx: Medications, supplements, HTN, cardiomyopathy, DM, cancer 9. FMhx: Cardiac dz - cardiomyopathies, MI, arrhythmias 10. Shx: recreational drugs, alcohol
17. Case 24 A 62-year-old man presents to your office for an acute visit because of coughing and shortness of breath. He is well known to you because of multiple office visits in the past few years for similar reasons. He has a chronic "smoker's cough" but reports that in the past 2 days his cough has increased, his sputum has changed from white to green in color, and he has had to increase the frequency with which he uses his albuterol inhaler. He denies having a fever, chest pain, peripheral edema, or other symptoms. His medical history is significant for hypertension, peripheral vascular disease, and two hospitalizations for pneumonia in the past 5 years. He has a 60 pack-year history of smoking and continues to smoke two packs of cigarettes a day. What PE would you perform? What diagnostic tests?
1. Respiratory exam - ausculatation, percussion, chest expansion test, look at chest (for barrel chest) 2. Cardiac exam - 3. HEENT: look at conjunctiva, look at throat, look in his nose 4. Skin: check for cyanosis, rash, check for edema 5. Labs - CBC (anemia, polycythemia, lymphocytosis), CMP (assess electrolytes, acid/base balance) 6. Spirometry- FEV1/FVC, Bronchodilator challenge, Chest x-ray (assess airway changes and cardiac abnormalities, EKG - changes due to COPD, cardiac cause and assess for ischemia due to COPD
Case 6 A 55-year-old man is brought into the emergency department (ED) by his wife for altered mental status (AMS). She states that for the past day, he has been confused and unsteady when he walks. The patient has a history of hypertension (HTN) and hyperlipidemia. He complains of headache and blurry vision. On examination, he is alert and oriented to person only. On fundoscopy, the optic discs appear hyperemic and swollen, with a loss of sharp margins. His neurological examination is nonfocal, and he otherwise has a normal physical examination. The patient's vital signs are a blood pressure of 245/140 mm Hg, heart rate of 95 beats per minute, respiratory rate of 18 breaths per minute, and oxygen saturation of 98% on room air, and he is afebrile. What is your working diagnosis and ddx? What's the next step for management?
1. Summary: A 55-year-old man with a history of HTN presents with AMS, headache, and blurry vision with a blood pressure of 245/140 mm Hg. His physical examination is significant for bilateral papilledema and AMS. 2. Most likely diagnosis: Hypertensive encephalopathy. 3. Ddx: Hemorrhagic or ischemic stroke, meningitis, encephalitis, mass lesion and toxic ingestion 4. Best management: Confirm the diagnosis by ruling out ischemic or hemorrhagic stroke, infection, and mass lesion. Lower blood pressure with intravenous (IV) medications and check for other evidence of end-organ damage.
Case 8 A 55-year-old man presents to the emergency department (ED) complaining of abdominal pain. The patient relates that he has been having intermittent pain throughout the abdomen for the past 12 hours, and since the onset of pain, he has vomited twice. His past medical history is significant for hypertension and colon cancer, for which he underwent laparoscopic right colectomy 8 months ago. The patient indicates that he has not had any recent abdominal complaints. His last bowel movement was 1 day ago, and he denies any weight loss and hematochezia. On physical examination, the patient is afebrile. The pulse rate is 98 beats per minute, blood pressure is 132/84 mm Hg, and respiratory rate is 22 breaths per minute. His cardiopulmonary examination is unremarkable. His abdomen is obese and mildly distended, with well-healed surgical scars. No tenderness, guarding, or hernias are noted. His bowel sounds are diminished, with occasional high-pitched sounds. The rectal examination reveals normal tone, empty rectal vault, and hemoccult-negative stool. What is on your ddx? What is your working dx?
1. Summary: A 55-year-old man with history of laparoscopic surgery 8 months previously for the resection of right colon carcinoma presents with intermittent abdominal pain and vomiting. The physical examination reveals no abdominal wall or groin hernias, no tenderness, and high-pitched bowel sounds. 2. Ddx: Intestinal ileus, adhesions, ischemia, and obstruction from recurrence of metastatic colon carcinoma. PUD, acute gastritis, biliary colic, acute cholecystitis, ascending cholangitis 3. Most likely diagnosis: Bowel obstruction. It is unclear whether the intestinal obstruction is involving the large or small bowel or whether it is complete or partial obstruction. 4. Next steps in evaluation: Diagnostic radiography, which can be either plain x-rays or a computed tomography (CT). 5. The presence of fever, tachycardia, persistent abdominal pain, abdominal tenderness, leukocytosis, and high-grade obstruction are associated with the increased likelihood of bowel necrosis. These findings should prompt early referral to a surgeon, and patients with these findings are more likely to benefit from early surgical interventions. ND plan: F/U in 2-4 weeks after release from surgeon or team, probiotics to replenish gut flora/microbiome, glutamine to help heal the tissues, aloe vera as anti-inflammatory and healing gut, vegetable broth, bone broth, smoothies w/protein powder to increase nutrients and help the healing process as well
Case 10 A 17-year-old adolescent male arrives at the emergency department (ED) after he developed an acute onset of severe right testicular pain about 4 hours ago while at soccer practice. The patient does not recall any recent trauma to the area and denies any fever, dysuria, or penile discharge. Although he has nausea, he does not have any abdominal pain or vomiting. On examination, his temperature is 99.5°F, blood pressure is 138/84 mm Hg, heart rate is 104 beats per minute, and respiratory rate is 22 breaths per minute. He is in acute distress due to pain. His abdomen is benign. On visual inspection, he has right scrotal erythema and swelling, although there are no penile lesions or discharge. Because his scrotum is so diffusely tender, it is difficult to examine it more closely. However, there is no testicular rise when his inner thigh is stroked. His urinalysis shows 3-5 white blood cells (WBCs)/high power field (hpf). What is your working diagnosis? What's on your ddx?
1. Summary: This is a 17-year-old adolescent male who presents with acute onset right testicular pain without any preceding trauma. 2. Most likely diagnosis: Testicular torsion. a. Ddx: Testicular torsion, epididymitis, orchitis, torsion of the testicular appendages, hernia, hydrocele, and testicular tumor. 3. Next diagnostic step: Urological consultation. Manual detorsion can be attempted while awaiting the consultant.
Case 26 A previously healthy 48-year-old man presents to his primary care office with severe low back pain that began the previous day after he helped his daughter move into her college dorm. What questions do you have for this patient?
1. Trauma, injury? 2. What is the character of the pain? 3. Does the pain radiate anywhere? 4. Scale of 1-10 how severe is the pain? 5. Does anything make it better? Worse? 6. Did he have a change in bowel movements? Change in urination? 7. Weakness in legs, numbness or tingling? 8. Fever, chills, weight loss, malaise? 9. How is it affecting his everyday life? 10. PMhx: medications, supplements, surgeries, procedures
Case 25 A 45-year-old man presents to the clinic with a cough productive of purulent sputum of 3 weeks' duration. What questions do you have for this patient?
1. Was he ill previously? Has anyone in his home been ill? 2. Has he been traveling recently? 3. What color is the sputum? Has it changed color over time? 4. Does he have SOB, wheezing, chest tightness? 5. Does he have nasal congestion, runny nose, sore throat? 6. Does he have fever, nausea, vomiting? 7. Change in bowel movements? 8. PMhx: medications, supplements, history of respiratory illnesses, allergies 9. Shx: smoking, recreational drugs, alcohol use,
Case 10 A 17-year-old adolescent male arrives at the emergency department (ED) after he developed an acute onset of severe right testicular pain about 4 hours ago while at soccer practice. What questions do you have for this patient?
1. Was he injured during practice? 2. Duration of pain? Alleviating and aggravating factors? Better in a different position? 3. Has this every happened before? 4. Does he have any N/V, fever, changes in urination? 5. Any abdominal pain, pelvic pain or penile pain? 6. Penile discharge, change in color of testes or penis?
Case 9 A 19-year-old woman is brought into the emergency department (ED) complaining of abdominal pain and diarrhea of 3-day duration. She has also been nauseous and has not been able to drink much liquid. Five days ago she returned from a camping trip in New Mexico but did not drink from natural streams. She denies fever but states that she has had some chills. Her stools have been watery, brown, and profuse. The patient denies health problems. On examination, the patient is thin and pale. Her mucous membranes are dry. Her temperature is 37.2°C (99°F), heart rate is 110 beats per minute, and blood pressure is 90/60 mm Hg. The skin has no lesions. Her heart and lung examinations are unremarkable except tachycardia. The abdominal examination reveals hyperactive bowel sounds and no masses. There is diffuse mild tenderness but no guarding or rebound. Rectal examination demonstrates no tenderness or masses and is hemoccult negative. The complete blood count reveals a leukocyte count of 16,000 cells/mm3. The pregnancy test is negative. What is your working dx? What is your ddx?
1. Wdx: Infectious diarrhea 2. Ddx a. Infectious: E. coli, Campylobacter, Shigella, Salmonella, Giardia, less likely due to non-bloody stools - Enteroinvasive E. coli species, Yersinia, Shigella and Entamoeba histolytica. b. Non-infectious 3. Most likely diagnosis: Acute volume depletion and possible electrolyte abnormalities secondary to acute diarrhea. 4. Next diagnostic step: Test stool for fecal leukocytes. 5. Next step in therapy: Intravenous fluid hydration. Antibiotic therapy may be indicated when symptoms do not resolve with supportive care and stool-reducing agents. 6. Most acute diarrheas are self-limited. 7. One should be cautious when assessing acute diarrhea in immunosuppressed, very young, or elderly patients. 8. Significant dehydration, grossly bloody diarrhea, high fever, and nonresponse after 48 hours are warning signs of possible complicated diarrhea. 9. In general, acute uncomplicated diarrhea can be treated with oral electrolyte-fluid solution with or without empiric ciprofloxacin
Case 8 A 55-year-old man presents to the emergency department (ED) complaining of abdominal pain. The patient relates that he has been having intermittent pain throughout the abdomen for the past 12 hours, and since the onset of pain, he has vomited twice. What questions do you have for this patient?
1. What happened before the pain? Any trauma, MVA, fall, someone hit them? 2. Where is the pain located? Does the pain radiate? 3. Does the pain change with movement, food or other triggers? 4. Fever/chills, weight loss, diarrhea, changes in bowel movements? Blood in stool, dark, tarry stools? 5. Back pain, changes in urination? 6. Medications/supplements? Recreational drug use? Alcohol use? 7. PMhx: SIBO, IBD, Celiac disease, diverticulitis, cholecystitis, pyelonephritis
Case 6 A 55-year-old man is brought into the emergency department (ED) by his wife for altered mental status (AMS). She states that for the past day, he has been confused and unsteady when he walks. What questions do you have for this patient?
1. What medications/supplements is he on? Herbs, NSAIDs, 2. Duration of prior HTN history 3. Neuro sx such as HA, sudden severe HA, N/V, dyspnea, orthopnea, cough (w/pinkish phlegm), visual disturbances, diaphoresis, dysphagia, weakness, numbness, or tingling of extremities. Bladder discomfort, hesitant urinary stream, and/or polyuria/oliguria/dysuria 4. PMhx: HTN, hyperlipidemia, DM, hx of sleep apnea 5. Shx: recreational drug use, alcohol and timing of last drink, oral contraceptives
Case 9 A 19-year-old woman is brought into the emergency department (ED) complaining of abdominal pain and diarrhea of 3-day duration. What are your questions?
1. Where is the abdominal pain located? Does it radiate? 2. How often is she going to the bathroom? 3. Has she noticed any blood, mucous, tarry stools? Describe stool, fluffy pieces, more watery? 4. Has she been out of the country, traveling, camping? Drank water that's not bottled, tap, stream, river? 5. What did she eat? Is she able to keep any foods or liquids down now? 6. Has she had fever/chills, N/V, headache, fatigue, confusion? 7. PMhx: medications, supplements, surgeries, GI disorders SIBO, PUD, 8.
Case 2 A 28-year-old man arrives at the emergency department (ED) complaining of 1 day of chest pain (CP) beginning suddenly after multiple episodes of retching and vomiting following a night of heavy drinking. What are your questions? What physical exam will you do?
1. Where is the location of the pain? 2. Does the pain radiate anywhere? 3. Experiencing palpitations? Having shortness or breath or difficulty breathing? 4. On any medication? Diagnosed w/HTN, high cholesterol, diabetes, etc.? 5. HEENT - look in throat, check neck, Cardio exam, pulmonary exam, abdominal exam - auscultation, percussion, palpation, EKG
Case 1 A 61-year-old man arrives at the emergency department (ED) complaining of chest pain. The pain began 45 minutes ago while watching television and is described as severe and pressure-like. What are your questions? What physical exam will you do?
1. Where is the pain located? 2. Does the pain radiate anywhere? 3. Are you experiencing shortness of breath or difficulty breathing? 4. Fevers? Sweatiness? 5. Do you have a history of HTN, high cholesterol, diabetes, MI? 6. Cardiac and pulmonary exam
Case 19 A 26-year-old woman presents to the emergency department (ED) with a 6-hour history of worsening abdominal pain. What are your questions?
1. Where is the pain located? Does the pain radiate anywhere? 2. Can she describe the pain? Superficial, dull, crampy, sharp? Is the pain constant, intermittent? 3. Severity of pain a. 0 - no pain b. 1 - 4 Advil or Tylenol c. 4 - 7 need pain killers d. 7 - 10 emergency room 4. Alleviating factors? Aggravating factor? Have you tried anything that helps? Do you notice anything that triggers the pain or makes it worse? (food, drinks, movements)? 5. Did anything occur prior to the pain? Trauma, infection, travel, food, drink, anyone else sick? 6. Fever, nausea, vomiting, headache? 7. Lightheadedness, dizziness, or syncope? 8. Change in urination increase in frequency, urgency, blood urine? Change in bowel movements diarrhea, constipation? 9. Does she have any vaginal discharge, bleeding, unusual odor? 10. Sexually active? 5 P's - partners, practices, pregnancy prevention, protection, past history of STDs? 11. Last menstrual cycle? 12. PMhx: medications, supplements, surgeries, any procedures? Contraceptives, IUD? History of STI, PID? 13. Shx: recreational drugs, alcohol use, smoking
5. Case 7 You are working in the emergency department (ED) of a 15-bed rural hospital without CT scan capabilities, and a 25-year-old, previously healthy woman presents for evaluation of abdominal pain. The patient describes her pain as having been present for the past 3 days. What are your next questions?
1. Where is the pain located? Does the pain radiate anywhere? 2. Describe the pain? Sharp, Dull, cramping, deep, aching, superficial? 3. How long does the pain last intermittent or constant? 4. Any triggers, food, movement? 5. Is there a history of trauma? Did she eat anything out of the ordinary? Recent travel? Camping? 6. Has this ever happened before? Any procedures - pregnancy termination 7. Is she sexually active? Does she have an IUD, oral contraceptives? Does she use protection? 8. Fever/chills, headache, N/V, vaginal discharge, itching, odor, urinary changes? Bowel changes? 9. Back pain, pain or bleeding with sex, bleeding btw menses? 10. Last menses?
Case 5 A 63-year-old woman arrives in the emergency department (ED) in respiratory distress. The paramedics who transported her were not able to obtain any information about her past medical history but did bring her bag of medications, which included furosemide. On examination, her temperature is 37.5°C (99.5°F), blood pressure is 220/112 mm Hg, heart rate is 130 beats per minute, respiratory rate is 36 breaths per minute, and oxygen saturation is 93% on a non-rebreather mask. What physical exam would you perform? What are your next diagnostic steps?
1. Working dx: Congenital heart failure w/pulmonary edema 2. Ddx: Asthma, COPD, pneumonia, pulmonary embolism, pulmonary edema, hypothyroidism 3. Summary: This is a 63-year-old woman in respiratory distress with signs of heart failure (HF) and fluid overload. 4. Most likely diagnosis: Congestive heart failure (CHF) and cardiogenic acute pulmonary edema. 5. Most appropriate next step: Management of the ABCs (airway, breathing, circulation), preload and afterload reduction, and eventual diuresis.
Case 11 A 54-year-old man is brought to the emergency department with complaints of generalized weakness, nausea, and nonspecific feelings of illness. The symptoms have progressed insidiously over 2-3 days. What questions would you like to ask?
1. Would ask about fever/chills, vomiting, weight loss? 2. Where is weakness (extremities, head), paralysis, cramping, numbness, tingling? 3. Ask about lightheadedness, dizziness, fainting? 4. Abdominal pain, changes in bowel movements (diarrhea, constipation), urinary changes (polyuria, dysuria, oliguria)? 5. Chest pain, pressure, palpitations? 6. Confusion, headaches, changes in vision? 7. PMhx - medications, supplements, surgeries, cardiovascular disease, DM, 8. SHx - alcohol use, recreational drug use
Case 23 A 28-year-old man presents to your office complaining of a 3-month history of rhinorrhea, itchy eyes, and exertional cough and wheezing. These symptoms have been progressively worsening over the past few months. His past medical history is significant for seasonal allergies to pollen and ragweed. His family history is significant only for hypertension in both parents. His siblings and children are in good health without allergies or respiratory illness. He does not smoke or use illicit drugs and only drinks alcohol rarely. He has worked as an animal laboratory technician for the last 6 months. On questioning, his symptoms were initially more severe toward the end of the work week but are now continuous. He has been taking over-the-counter antihistamines, which helped initially but no longer relieve his allergic symptoms. On review of systems, he has noted hives that are less prominent now that he has been taking the antihistamines on a regular basis. On examination, he is afebrile, his body mass index is 23 kg/m2, blood pressure is 120/75 mm Hg, pulse is 72 beats/min, and respiratory rate is 18 breaths/min. What PE would you like to perform? What diagnostic test do you want to do?
11. HEENT: Check eyes (conjunctiva), check nose, check throat for racing stripes 12. Cardiac exam 13. Respiratory exam 14. Spirometry
Case 26 A previously healthy 48-year-old man presents to his primary care office with severe low back pain that began the previous day after he helped his daughter move into her college dorm. He denies any trauma or previous back injury. He describes the pain as generally "achy" and sometimes characterized as being "sharp" when he moves suddenly. The pain is located in his lower back and radiates down the back of both legs to the middle of his posterior thighs. He denies any bladder or bowel incontinence or weakness in his legs. He denies fever, chills, weight loss, or malaise. He finds it very difficult to stand for prolonged periods of time because he cannot find a comfortable position. He states that this is the worst back pain he has ever experienced. It has not been relieved with acetaminophen or ibuprofen. His past medical history is significant for hypertension, and his only medication is lisinopril daily. He does not smoke or use illicit drugs and only drinks alcohol on occasion. What PE would you like to perform? What other diagnostic tests would you like to do?
11. MSK exam ROM, straight leg raise, Kemp's test, strength testing, DTR, Trendelenburg - weak gluteal muscle
Case 20 A 34-year-old man presents to the emergency department (ED) complaining of shortness of breath and right-sided chest pain that increases with deep breathing. He states that it started suddenly when he woke up and worsened with activity. He denies fever, chills, nausea, vomiting, or cough. He has a recent history of multiple gunshot wounds, resulting in ongoing pain in his upper back and T10 paraplegia. He quit smoking tobacco when he was hospitalized and denies any alcohol or illicit drug use. One week ago, he was discharged from the hospital to a rehabilitation facility. He is currently taking acetaminophen/hydrocodone and ibuprofen for his pain, which has increased with physical therapy and occupational therapy. He is also taking hydrochlorothiazide and lisinopril for hypertension and fluoxetine for depression. On physical examination, he is an otherwise fit young man who appears slightly short of breath and uncomfortable. His heart rate is 101 beats per minute, blood pressure is 110/78 mm Hg, and respiratory rate is 26 breaths per minute. His pulse oximetry is 96% on 2 L of O2 by nasal cannula. His lungs are clear to auscultation. There is mild swelling of his left calf. He has no sensation in his lower extremities. Laboratory studies reveal a white blood cell count of 10,000 cells/mm3. Hemoglobin, hematocrit, electrolytes, and renal function are all within normal limits. A 12-lead electrocardiogram (ECG) reveals a sinus rhythm at a rate of 103 beats per minute. His chest radiograph reveals minimal bibasilar atelectasis but no evidence of infiltrates or effusions. What's on your ddx? What's working dx? What's your plan?
12. Asthma, pulmonary edema, pneumonia, bronchitis, heart failure, MI, unstable or stable angina, COPD 13. Summary: A 34-year-old man presents with - An acute onset of right-sided pleuritic chest pain and dyspnea, No fever, chills, nausea, vomiting, or cough, T10 paraplegia, Past medical history of hypertension and depression, Swelling of the left calf, Normal lab and chest x-ray findings 14. Most likely diagnosis: Pulmonary embolism (PE) secondary to deep venous thrombosis (DVT) in the left lower extremity. 15. Next diagnostic steps: For evaluation of PE/DVT, D-dimer level, venous duplex ultrasonography, ventilation-perfusion scan, pulmonary computed tomography angiography (recommended for dx PE), and catheter pulmonary angiography are available and may be applied on a selective basis.
Case 26 A previously healthy 48-year-old man presents to his primary care office with severe low back pain that began the previous day after he helped his daughter move into her college dorm. He denies any trauma or previous back injury. He describes the pain as generally "achy" and sometimes characterized as being "sharp" when he moves suddenly. The pain is located in his lower back and radiates down the back of both legs to the middle of his posterior thighs. He denies any bladder or bowel incontinence or weakness in his legs. He denies fever, chills, weight loss, or malaise. He finds it very difficult to stand for prolonged periods of time because he cannot find a comfortable position. He states that this is the worst back pain he has ever experienced. It has not been relieved with acetaminophen or ibuprofen. His past medical history is significant for hypertension, and his only medication is lisinopril daily. He does not smoke or use illicit drugs and only drinks alcohol on occasion. On physical examination, he is well developed, overweight, and in moderate discomfort. His vitals are within normal limits. On neuromuscular examination, he has moderate tenderness bilaterally in his lumbar paraspinous muscles, and his lumbar flexion and extension are limited by pain. Strength and sensation are within normal limits and are symmetrical bilaterally. He has normal and symmetric knee and ankle deep tendon reflexes. Straight leg raise testing is negative bilaterally, and gait is within normal limits. What is on your ddx? What is your working dx? What is your plan?
12. Ddx: Lumbar sprain or strain, sciatica, spinal stenosis, spondylosis (degeneration arthritis) osteoarthritis, Red flag cauda equina 13. Wdx: Lumbar sacral sprain 14. Plan: Take tylenol or acetominophen pain, Antiinflammatory herbs (Turmeric, Boswellia, BCQ) to reduction inflammation of the back muscles, Topical magnesium to relax the muscles, drink plenty of water to stay hydrated to make sure your muscles can heal properly allow proper bloodflow to bring nutrients to the area and remove 15. Summary: A 48-year-old man presents with, Acute onset of low back pain after strenuous activity, Previously healthy status, Appears overweight, unremarkable neurologic examination, Denial of any systemic complaints, Past medical history significant for hypertension, with his only medication being lisinopril daily 16. Most likely diagnosis: Acute low back pain, lumbar sacral strain. 17. Most appropriate workup: No formal workup is required unless symptoms persist after conservative treatment for at least 1 month. 18. Best treatment plan: Relative rest (but not bed rest), nonsteroidal anti-inflammatory drugs (NSAIDs) or acetaminophen, and muscle relaxants. 19. This clinical scenario is best managed by symptomatic therapies for 4 to 6 weeks without imaging and with close follow-up if symptoms do not resolve. In the majority of patients with acute back pain in the absence of red flag symptoms, laboratory tests and imaging studies are not required. Education in proper lifting techniques and exercise therapy to improve core and lumbar sacral strength and flexibility may help to prevent future strain and injury.
Case 23 A 28-year-old man presents to your office complaining of a 3-month history of rhinorrhea, itchy eyes, and exertional cough and wheezing. These symptoms have been progressively worsening over the past few months. His past medical history is significant for seasonal allergies to pollen and ragweed. His family history is significant only for hypertension in both parents. His siblings and children are in good health without allergies or respiratory illness. He does not smoke or use illicit drugs and only drinks alcohol rarely. He has worked as an animal laboratory technician for the last 6 months. On questioning, his symptoms were initially more severe toward the end of the work week but are now continuous. He has been taking over-the-counter antihistamines, which helped initially but no longer relieve his allergic symptoms. On review of systems, he has noted hives that are less prominent now that he has been taking the antihistamines on a regular basis. On examination, he is afebrile, his body mass index is 23 kg/m2, blood pressure is 120/75 mm Hg, pulse is 72 beats/min, and respiratory rate is 18 breaths/min. His conjunctivae are injected, there is mild clear ocular discharge, and his nasal turbinates are boggy without visible polyps. His lung examination reveals a prolonged expiratory-to-inspiratory ratio and end-expiratory wheezing at the bilateral bases. His heart examination is unremarkable, and there is no peripheral edema. What is on your ddx? What is your working dx? What is your plan?
15. Ddx: Bronchitis, asthma, bronchiectasis, Heart failure, COPD (chronic bronchitis or emphysema), allergic rhinitis and sinusitis, 16. Wdx: Asthma 17. Plan: 18. Summary: A 28-year-old man presents with, Classic signs and symptoms of asthma, A constellation of ocular, nasal, and pulmonary symptoms temporally related to work and environmental conditions, which is suspicious for occupational-related asthma and allergy, Injected conjunctivae, mild clear ocular discharge, and boggy nasal turbinates without visible polyps, Lung examination revealing a prolonged expiratory-to-inspiratory ratio and end-expiratory wheezing at the bilateral bases 19. Most likely diagnosis: Occupation-related allergic asthma, allergy mediated. 20. Further evaluation: Peak flow measurements pre- and post-beta-agonist treatment. Asthma dx confirmed by improvement in FEV1 >= 12% w/bronchodilator. Further workup should include a chest radiograph if concerned about alternative etiologies, pulmonary function testing with bronchodilator challenge, and consideration of allergen testing. 21. Initial steps in therapy: Initial treatment with a short-acting inhaled beta-agonist (SABA) such as albuterol. A short course of oral steroids should be considered if the patient continues to have wheezing, decreased pulse oximetry, and decreased predicted peak flow measurements after beta-agonist therapy. Antihistamines should be considered for allergic symptoms. This patient should be removed from his current duties of working with laboratory animals.
Case 21 A 63-year-old woman presents to the emergency department (ED) with severe left eye pain, redness, and blurred vision for 3 hours. Her right eye is asymptomatic. She denies preceding trauma, photophobia, ocular discharge, increased tearing, prior similar events, or past eye surgery. She is farsighted and sometimes wears nonprescription reading glasses. Other symptoms include seeing colored halos around the light fixtures in the ED, having a headache over her left brow, some nausea, and one episode of vomiting. She denies dizziness, weakness, imbalance, abdominal pain, and chest pain. On examination, her blood pressure is 138/80 mm Hg, and the other vital signs are normal. She is alert and in obvious discomfort but can tolerate ambient light. She has no periorbital signs of trauma. The left conjunctiva has ciliary flush (circumferential reddish ring around the cornea) but no discharge or visible foreign body. Visual acuity is 20/30 in the right eye but only finger counting in the left eye. Visual fields are grossly intact. Gentle palpation of the closed left eye reveals that it is much firmer than the right. Her left pupil is 5 mm, fixed, and unreactive. Her right eye appears normal, the pupil is 3 mm and briskly reactive. She does not experience pain in the left eye when direct light is applied to the right eye (absent consensual photophobia). When a penlight is shone temporally across each eye, the beam does not reach the nasal side. Extraocular movements are intact and nonpainful. The left cornea is slightly cloudy, which makes fundoscopy difficult. The right fundus appears normal. Her temporal arteries are pulsatile and nontender. The rest of the physical examination, including the remainder of the neurologic examination, is normal. What is on your ddx? What's your wdx? What is your plan?
2. Acute angle glaucoma, viral or bacterial conjunctivitis, migraine, cerebrovascular event, anterior uveitis, herpes zoster ophthalmicus, scleritis, eye trauma/chemical exposure a. In addition to AACG, vision-threatening and painful causes of a red eye include severe conjunctivitis, keratitis, corneal ulcer, anterior uveitis, endophthalmitis, orbital cellulitis, scleritis, and temporal arteritis 3. Wdx: acute angle glaucoma 4. Summary: A 63-year-old woman presents with, Acute onset of left eye redness, pain, and markedly decreased visual acuity, A left eye that feels firmer to palpation compared to the right eye, A cloudy left cornea with a fixed and dilated pupil, Next diagnostic step: Measure intraocular pressures (IOP) in both eyes using tonometry and perform a slit-lamp examination to assess for inflammatory changes and the presence of red blood cells or white cells in the anterior chamber. Most likely diagnosis: Acute angle-closure glaucoma.Best next therapeutic step: Preserve vision by lowering the IOP as quickly as possible. Consult ophthalmology emergently. After referral: annual eye examination, low fat diet, supplement zinc and avoid caffeine
Case 15 A 57-year-old man presents to the emergency department (ED) with a 1-month history of worsening low back pain. The pain radiates down the back of both legs and suddenly increased yesterday. For the past 2 days, the patient has been having difficulty voiding and has had "to force the urine out." He has also noticed that the skin around his anus feels numb when he wipes with toilet tissue. He works in a warehouse but has been on light duty for the past month due to his back pain. He denies prior trauma to or surgery on his back. What is on your ddx? What is your wdx?
3. Lumbar strain/sprain, Multiple Myeloma, Cauda equina, Lumbar radiculopathy a. he ED physician must consider the "cannot miss" diagnoses: cauda equina syndrome, spinal fracture, spinal infection (epidural abscess or spondylitis), and malignancy 4. Summary: A 57-year-old warehouse worker has a 1-month history of worsening low back pain with radiation bilaterally to his legs. The pain increased suddenly and is now associated with perianal numbness and difficulty voiding. He denies trauma or prior surgery. 5. Most likely diagnosis: Cauda equina syndrome (CES). 6. Next diagnostic step: STAT magnetic resonance imaging (MRI) of the lumbar and sacral spine, since this is a potential surgical emergency
Case 4 A 25-year-old man presents to the emergency department (ED) with palpitations and lightheadedness. These symptoms started acutely about 1 hour prior to arrival while he was watching television. The patient does not have any chest pain or shortness of breath. He also denies any recent fever, upper respiratory symptoms, and hemoptysis. He does not have any significant past medical history or family history. He is not taking any medications, does not smoke, and has never used any illicit drugs. On examination, his temperature is 98.2°F, blood pressure is 88/46 mm Hg, heart rate is 186 beats per minute, respiratory rate is 22 breaths per minute, and oxygen saturation is 97% on room air. He is mildly anxious but otherwise in no acute distress. He does not have any jugular venous distention. His lungs are clear to auscultation, and his heart sounds are regular without any murmurs, rubs, or gallops. There is no lower extremity edema, and peripheral pulses are equal in all four extremities. The cardiac monitor reveals a regular rhythm with narrow-QRS complexes at a rate of 180 to190 beats per minute. 1. What is your working diagnosis? 2. What is on your ddx?
3. Summary: This is a 25-year-old man with acute onset of palpitations and dizziness. He is hypotensive and has a narrow-QRS complex tachycardia at a rate of 180 to 190 beats per minute. 4. Most likely diagnosis: Supraventricular tachycardia (SVT). a. Ddx: Physiological sinus tachycardia, atrial flutter, atrial fibrillation, hyperthyroidism, dehydration 5. Most appropriate next step: Obtain IV access and a 12-lead ECG. Prepare for synchronized cardioversion of this unstable patient with a tachyarrhythmia.
Case 3 A 73-year-old woman presents to the emergency department complaining of palpitations and new shortness of breath with minor exertion for the past 2 weeks. Previously, she could walk everywhere, but now she becomes fatigued climbing the stairs of her home. Occasionally, she has felt her heart racing even when she is at rest. Her past medical history is notable for diet controlled diabetes and hypertension, for which she takes hydrochlorothiazide and amlodipine. On physical examination, she appears comfortable and speaks in full sentences without difficulty. Her blood pressure is 130/90 mm Hg, heart rate is 144 beats per minute, respiratory rate is 18 breaths per minute, oxygen saturation is 98% on room air, and temperature is 37°C (98.6°F). The head and neck examination is unremarkable. Her lungs are clear to auscultation. Her heartbeat is irregular and rapid, without murmurs, rubs, or gallops. She has no extremity edema or jugular venous distension. Her abdomen is soft and nontender, without masses. Labs show a normal complete blood count (CBC), normal electrolytes, blood urea nitrogen (BUN), creatinine, troponin, brain natriuretic peptide (BNP), and thyroid stimulating hormone. A chest x-ray reveals a normal cardiac silhouette with no pulmonary edema. The ECG is shown below (Figure 9-1). 1. What is the most likely diagnosis? 2. What are some of the most common contributing factors? 3. What are some complications?
4. Most likely diagnosis: Atrial fibrillation (AF) with rapid ventricular response (RVR). 5. Common contributing factors: Increasing age, underlying cardiopulmonary disease (such as hypertension, heart failure, valvular disease, and chronic obstructive pulmonary disease [COPD]), hyperthyroidism, sepsis, pulmonary embolism, and electrolyte abnormalities. 6. Complications: Early—diminished cardiac output (CO) and hypotension. Late—thromboembolism and cardiomyopathy. 7. ND plan - referral to cardiologist, F/U cardioprotective herbs (hibiscus, hawthorne), diet -
Case 6 A 55-year-old man is brought into the emergency department (ED) by his wife for altered mental status (AMS). She states that for the past day, he has been confused and unsteady when he walks. The patient has a history of hypertension (HTN) and hyperlipidemia. He complains of headache and blurry vision. What physical exam should we do? What other diagnostic tests would you do?
6. Head and neck: Assess for head injury, auscultate carotid arteries for bruits, evaluate for JVD 7. Neuro - assess for facial weakness or dysarthria, (stupor, seizures, delirium, agitation, MOCA and MME) 8. Cardio exam (palpation apical pulse assess enlargement, auscultation - murmurs and S3 and S4 gallops), peripheral pulses and edema 9. Pulmonary exam (auscultation - wheezing, crackles, rales - pulmonary edema) 10. Eye exam: fundoscopic exam (papilledema, retinal hemorrhages, exudates) 11. Abdominal exam: evaluate for AAA, bruits suggest renal artery stenosis 12. Psych: orientation, 13. UA (check for proteinuria, RBC, RBC casts may point toward renal failure or glomerulonephritis), CBC (check for microangiopathic anemia), 14. CMP (creatinine and BUN) , blood glucose (hypoglycemia) 15. EKG - check for myocardial ischemia, infarction, and/or signs of left ventricular hypertrophy. BNP and chest x-ray to rule out heart and acute pulmonary edema 16. Non-contrast Head CT to rule out mass lesion and hemorrhagic or ischemic stroke
Case 2 A 28-year-old man arrives at the emergency department (ED) complaining of 1 day of chest pain (CP) beginning suddenly after multiple episodes of retching and vomiting following a night of heavy drinking. Since the pain began, he has had subjective fevers, feels very weak, is unable to tolerate any food or water, and has urinated only once. He describes lower anterior CP that is non-radiating and of moderately severe intensity, aggravated by swallowing. While he is speaking in full sentences, he states that he has to control his breathing to avoid a "tight feeling in his throat," and he is concerned that his voice sounds "different." The review of systems is otherwise negative. His temperature is 38°C, heart rate is 132 beats per minute, blood pressure is 148/74 mm Hg, and respiration rate is 22 breaths per minute with an O2 saturation of 98% on room air. Physical examination shows an ill and uncomfortable appearing man who is tachycardic, diaphoretic, and borderline febrile. He has dry mucous membranes and faint bibasilar crackles. He has no past medical history and is taking no medications. 6. What is your working diagnosis? 7. What are the next diagnostic steps? 8. What are the initial therapies?
9. Most likely diagnosis: Spontaneous esophageal perforation or Boerhaave syndrome. a. Ddx: MI, pulmonary embolism, dissecting aortic aneurysm, pancreatitis 10. Next management steps: Place the patient on a cardiac monitor, establish IV access and obtain an electrocardiogram (ECG) immediately. A chest x-ray (CXR) should be obtained as soon as possible. 11. Immediate therapies: Nil per os (NPO), nasogastric tube (NGT), IV resuscitation with isotonic fluids, broad spectrum antibiotics, pain management, and urgent surgical consultation. ND plan - F/U 2-4 weeks after healing from surgery, whenever released by surgeon or surgical team, glutamine 2 g powder qd to heal esophageal tissue, aloe vera get anti-inflammatory, soothing and healing tissues, vegetable broth, smoothies to maintain nutrients if diff swallowing, immune support herbs to help clear any infections, and probiotics to regrow or replenish gut flora/microbiome
Case 1 It is substernal in location, radiates to the jaw and left shoulder, and is associated with shortness of breath. The patient has hyperlipidemia and hypertension and takes simvastatin and hydrochlorothiazide. His blood pressure is 160/110 mm Hg, pulse rate is 93 beats per minute, respiration rate is 22 breaths per minute, temperature is 37.1°C, and oxygen saturation is 97%. The patient appears anxious, is diaphoretic and vomited once. On auscultation, faint crackles are heard at both lung bases. The cardiac and abdominal examinations are unremarkable. - 1. What is your working diagnosis? 2. What are the next diagnostic steps? 3. What are the initial therapies?
Answer 1. Myocardial infarction. Ddx: Angina pectoris, pericarditis, thoracic aortic dissection 2. Run EKG, IV access, chest x-ray, check serum calcium markers Troponin I, CKMB and myogloblin 3. Sending to the ED - Aspirin is the most important immediate therapy. Oxygen and sublingual nitroglycerin are also standard early therapies. Depending on the result of the ECG, emergency reperfusion therapy, such as a percutaneous coronary intervention (PCI), may be indicated. Intravenous beta-blockers, IV nitroglycerin, low-molecular-weight heparin, and additional antiplatelet agents, such as clopidogrel, might also be indicated.
Case 3 A 73-year-old woman presents to the emergency department complaining of palpitations and new shortness of breath with minor exertion for the past 2 weeks. Previously, she could walk everywhere, but now she becomes fatigued climbing the stairs of her home. Occasionally, she has felt her heart racing even when she is at rest. Her past medical history is notable for diet controlled diabetes and hypertension, for which she takes hydrochlorothiazide and amlodipine. On physical examination, she appears comfortable and speaks in full sentences without difficulty. Her blood pressure is 130/90 mm Hg, heart rate is 144 beats per minute, respiratory rate is 18 breaths per minute, oxygen saturation is 98% on room air, and temperature is 37°C (98.6°F). The head and neck examination is unremarkable. Her lungs are clear to auscultation. Her heartbeat is irregular and rapid, without murmurs, rubs, or gallops. She has no extremity edema or jugular venous distension. Her abdomen is soft and nontender, without masses. 1. What is your working diagnosis? 2. What are the next diagnostic steps?
Wdx 1. Atrial fibrillation, Ddx - hyperthyroidism, dehydration, anxiety Lab 2. EKG (check cause of SOB, palpitations), transthoracic and transesophageal echo 3. Blood tests - TSH, CMP, CBC, BNP, creatinine, troponin
Case 16 A 19-year-old man is brought in with diffuse abdominal pain and vomiting. The patient's symptoms began several days ago when he complained of "the flu." What questions do you want to ask?
• Does he have a fever? • Where is the pain located? • Does the pain radiate anywhere else? Duration of pain constant, intermittent? • What is the severity of pain? • Has anything ever happened like this before? • What symptoms did he have with the flu? • Anything make it better? Anything make it worse? • Any headache, fever, malaise, fatigue? • Does he have an increase in thirst or hunger? • Any urinary changes, bowel movement changes, pain w/urination? • PMhx: medications, supplements, surgeries etc. • Shx: recreational drug use, alcohol use, smoking • Fhx: autoimmune disease - thyroid, diabetes, cardiovascular, cancer
Case 14 A 50-year-old woman presents with a severe headache of abrupt onset of 10 hours' duration. The pain is diffuse, throbbing, and worsened when she went outside into the sunlight. She denies any recent fever, neck pain, numbness, weakness, vomiting, and any change in vision. She was concerned because she has never had a headache as strong as this before. Her past medical and family histories are unremarkable. She does not take any medications, does not smoke, and only drinks alcohol socially. On examination, her temperature is 36.9°C (98.4°F), blood pressure is 136/72 mm Hg, heart rate is 88 beats per minute, and respiratory rate is 16 breaths per minute. What PE do you want to perform? What other diagnostic tests do you want to do?
• HEENT - fundoscopic exam, PERRLA • MSK - ROM neck • Brudzinki's exam - • Neuro exam - cranial nerves, strength, reflexes, light and soft touch, finger to nose • CT scan